LSAT and Law School Admissions Forum

Get expert LSAT preparation and law school admissions advice from PowerScore Test Preparation.

User avatar
 Dave Killoran
PowerScore Staff
  • PowerScore Staff
  • Posts: 5853
  • Joined: Mar 25, 2011
|
#47012
Complete Question Explanation
(The complete setup for this game can be found here: lsat/viewtopic.php?t=17223)

The correct answer choice is (A)

If F serves in a given year then you can only have one more lawmaker. If G is the other lawmaker then V cannot serve that year, and since you must have either I or V, then I would have to serve. However that would result in three lawmakers—F, G, and I—which violates the conditions of the scenario. Thus, G cannot serve in the same year as F, and answer choice (A) is correct.

Get the most out of your LSAT Prep Plus subscription.

Analyze and track your performance with our Testing and Analytics Package.